1

$\def\k{\operatorname k}\def\W{\operatorname W}$

Using a Dirac $\delta(x)$ Fourier series and this post

$$\begin{align} y\cot(y)=\frac1x\implies\frac1{\sec^2(y)-x}=\int_0^\frac\pi2\delta(\tan(t)-xt)dt-\frac1{2|x-1|}=\frac14+\frac1\pi\sum_{n=1}^\infty\int_0^\frac\pi2\cos(n (\tan(t)-xt))dt-\frac1{2|x-1|}\end{align}$$

Now remember the Bateman k$_v(z)$ and Whittaker W$_{a,b}(x)$ functions:

$$\k_v(x)=\frac{\W_{\frac v2,\frac12}(2x)}{\left(\frac v2\right)!}=\frac2\pi\int_0^\frac\pi2\cos(x\tan(t)-vt)dt$$

Therefore, we hopefully solve for the smallest positive solution:

$$\frac{\tan(y)}y=x\implies y=\sec^{-1}\left(\sqrt{\frac1{\frac14+\frac12\sum\limits_{n=1}^\infty\frac{\W_{\frac{nx}2,\frac12}(2n)}{\left(\frac{nx}2\right)!}-\frac1{2|x-1|}}+x}\right)= \sec^{-1}\left(\sqrt{\frac1{\frac14+\frac12\sum\limits_{n=1}^\infty\k_{nx}(n)-\frac1{2|x-1|}}+x}\right) \tag1$$

There appears to be no other way to use the Bateman function to solve $\frac{\tan(y)}y=x$. A numerical tester with $y\to\sec^{-1}\left(\sqrt{\frac1y+x}\right)$ is inconclusive. Even though the above series is slow, it still could be a solution.

Is $(1)$ true?

Тyma Gaidash
  • 12,081
  • As I mentioned in a comment on my answer to the linked question, using the Fourier series representation of the Dirac comb in the integral $\int\limits_0^\frac\pi2\delta(\tan(t)-x,t),dt$ is problematic since $\underset{t\to \left(\frac{\pi }{2}\right)^-}{\text{lim}}(\tan (t)-x, t)=\infty$. – Steven Clark Feb 14 '23 at 16:09
  • @StevenClark It seems problematic, but numerical tests show the sum at least converges near the root of $\tan(t)-xt$. It is possible the sum is very slow, as Fourier series for roots sometimes are. – Тyma Gaidash Feb 14 '23 at 17:02
  • In the integral $\int\limits_0^\frac\pi2 \operatorname{\text{Ш}}{2 \pi}(\tan(t)-x, t),dt$, I believe the teeth close to $t=0$ contribute more than the teeth close to $t=\frac\pi2$ since the slope of $\tan(t)-x, t$ is much smaller for $t$ close to zero than it is for $t$ close to $\frac\pi2$, but I don't see how you can obtain an exact result using an analytic representation of $\operatorname{\text{Ш}}{2 \pi}(y)$ instead of an analytic representation of $\delta(y)$ in the integral. – Steven Clark Feb 14 '23 at 17:41
  • @StevenClark The added value from the $t=0$ root is discussed here, where you commented before, and is $\frac1{2|x-1|}$, which is subtracted in the question to only include the smallest positive zero from $\int_0^\frac\pi2\delta(\tan(t)-xt)dt-\frac1{2|x-1|}$ – Тyma Gaidash Feb 14 '23 at 19:09
  • Ok, thanks, I missed that. I'm wondering if a result can be derived using the analytic representation for $\delta(x)$ which I defined in formula (14) (which is the last formula) of the most recent answer I posted and accepted at https://mathoverflow.net/q/418502 to one of my own questions. – Steven Clark Feb 14 '23 at 20:12
  • Using $\delta(w)=\frac2\pi\int_0^\infty \cos(w t)dt$ is much less accurate than the sum even though it is like taking $T\to\infty$. – Тyma Gaidash Feb 21 '23 at 00:18

1 Answers1

3

The answer to your question is no. Any approach based on the integral

$$\int\limits_0^\frac\pi2\operatorname{\text{Ш}}_{2 \pi}(f_x(t))\,dt\tag{1}$$

of the Fourier series for the Dirac comb

$$\operatorname{\text{Ш}}_{2 \pi}(t)=\frac{1}{2 \pi}\underset{N\to\infty}{\text{lim}}\left(\sum\limits_{n=-N}^N e^{i n t}\right)=\frac{1}{2 \pi}+\frac{1}{\pi}\underset{N\to\infty}{\text{lim}}\left(\sum\limits_{n=1}^N \cos(n t)\right)\tag{2}$$

where

$$f_x(t)=\tan(t)-x t\tag{3}$$

is at best an approximation and not an exact formula. You may be able to compensate for the two contributions of the tooth of the Dirac comb at $t=0$, but all teeth in the Dirac comb in the right-half plane contribute to the end result.


Noting the Dirac comb $\operatorname{\text{Ш}}_{2 \pi}(t)$ is $2\pi$ periodic as illustrated in Figure (1) below, the following table illustrates the contribution of the tooth at $t=0$ and the first 5 teeth in the right-half plane for the case $f_2(t)=\tan(t)-2 t$ (corresponding to $x=2)$.

$$\begin{array}{cc} k & \int\limits_0^{\frac{\pi }{2}} \delta(f_2(t)-2 \pi k) \, dt \\ 0 & 0.225523 \\ 1 & 0.0119339 \\ 2 & 0.00413686 \\ 3 & 0.00208941 \\ 4 & 0.00125876 \\ 5 & 0.000840887 \\ \end{array}$$

Note the row for $k=0$ in the table above gives the correct result since Mathematica ignores the partial contribution of an analytic representation at $t=0$. Also note the contribution of each successive tooth in the right-half plane decreases in magnitude which is because the slope of $f_2(t)=\tan(t)-2 t$ increases rapidly as $t$ approaches $\frac{\pi}{2}$.


Illustration of formula (2)

Figure (1): Illustration of formula (2) for $\operatorname{\text{Ш}}_{2 \pi}(t)$ evaluated at $N=20$


Figure (2) below illustrates $f_2(t)=\tan(t)-2 t$ in blue and $\delta(f_2(t))$ in orange using the limit representation

$$\delta(t)=\underset{N\to\infty}{\text{lim}}\left(2 N\, \text{sinc}(2\, \pi N\, t)\right)\tag{4}$$

evaluated at $N=20$. Note $\delta(f_2(t))$ has a $\delta$-function spike at $t=0$ as well as at $t\approx 1.16556$ corresponding to the two zeros of $f_2(t)$.


Illustration of f_2(t) in blue and formula (4) for delta(f_2(t)) in orange

Figure (2): Illustration of $f_2(t)=\tan(t)-2 t$ in blue and formula (4) for $\delta(f_2(t))$ in orange


Figure (3) below illustrates $f_2(t)=\tan(t)-2 t$ in blue and $\delta(f_2(t)-2 \pi)$ in orange which corresponds to the first tooth of the Dirac comb in the right-half plane using the limit representation of $\delta(t)$ defined in formula (4) above evaluated at $N=20$. Note the $\delta$-function spike where $f_2(t)$ evaluates to $2 \pi$ (corresponding to the horizontal green reference line).


Illustration of f_2(t) in blue and formula (4) for delta(f_2(t)-2 pi) in orange

Figure (3): Illustration of $f_2(t)=\tan(t)-2 t$ in blue and formula (4) for $\delta(f_2(t)-2 \pi)$ in orange


Every time $f_x(t)$ crosses an integer multiple of $2 \pi$ the integral $\int\limits_0^\frac\pi2\operatorname{\text{Ш}}_{2 \pi}(f_x(t))\,dt$ will pick up another contribution of a tooth of the Dirac comb, and since $\underset{t\to \left(\frac{\pi }{2}\right)^-}{\text{lim}}(f_x(t))=\infty$ this includes a contribution from every tooth of the Dirac comb in the right-half plane.


As $x$ grows larger, the integral $\int\limits_0^\frac\pi2\operatorname{\text{Ш}}_{2 \pi}(f_x(t))\,dt$ will also begin to pick up contributions from teeth of the Dirac comb $\operatorname{\text{Ш}}_{2 \pi}(t)$ in the left-half plane. For example, Figure (4) below illustrates $f_{10}(t)=\tan(t)-10 t$ in blue and $\delta(f_{10}(t)+2 \pi)$ in orange which corresponds to the first tooth of the Dirac comb in the left-half plane using the limit representation of $\delta(t)$ defined in formula (4) above evaluated at $N=20$. Note the two $\delta$-function spikes where $f_{10}(t)$ evaluates to $-2 \pi$ (corresponding to the horizontal green reference line).


Illustration of f_{10}(t)=\tan(t)-10 t in blue and formula (4) for delta(f_{10}(t)+2 \pi) in orange

Figure (4): Illustration of $f_{10}(t)=\tan(t)-10 t$ in blue and formula (4) for $\delta(f_{10}(t)+2 \pi)$ in orange

Steven Clark
  • 7,363
  • Your answer will be accepted and bountied if there are no others. You showed the teeth interfere with the integral value and an online Pari/GP calculated that the $n=3000$ partial sum gives $\tan(x)=2x\implies x\approx 1.148$ when the correct result is $1.165$. Therefore, the series solution for the root does not work. Using https://mathoverflow.net/q/418502 may not work either for these reasons. – Тyma Gaidash Feb 19 '23 at 22:03
  • @TymaGaidash The formula I posted on MathOverflow is an analytic representation of the Dirac delta function $\delta(x)$, not the Dirac comb, but since the parameter $f_x(t)$ of $\int\limits_0^{\frac{\pi }{2}} \delta(f_x(t)),dt$ takes on all positive values it seems to me this series representation would need to be evaluated over the entire range $\sum\limits_{n=1}^\infty (...)$. The other problem with that formula is the $x\pm 1$ in two of the $\cos$ functions which I don't know how to handle in your approach above. – Steven Clark Feb 19 '23 at 22:44
  • @TymaGaidash It's too bad there doesn't seem to be a way to derive a result using a limit representation for $\delta(x)$ such as formula (4) in my answer above (except via numerical integration which I suspect is not particularly accurate due to the highly oscillatory nature of the limit representation which increases as the limit variable $N$ increases). – Steven Clark Feb 19 '23 at 22:48
  • 1
    @TymaGaidash Assuming your derivation of the series representation in your answer above is correct, then you can probably increase the accuracy of your approach by increasing the period $T$ of the Dirac comb $\operatorname{\text{Ш}}T(t)=1+2 \underset{N\to\infty}{\text{lim}}\left(\sum\limits{n=1}^N \cos\left(\frac{2 \pi n t}{T}\right)\right)$ to push the teeth in the right-half plane out such that the slope of $\tan(t)-x t$ is much greater and therefore the undesirable contribution of these teeth will be much smaller. – Steven Clark Feb 19 '23 at 23:52
  • @TymaGaidash But I believe the disadvantage of this approach is as you increase the period $T$ of the Dirac comb $\operatorname{\text{Ш}}_T(t)$, you'll also need to increase the number of terms you evaluate in your series representation. – Steven Clark Feb 19 '23 at 23:53
  • There may be factor of $\frac2T$ for the Fourier cosine series of $\unicode{1064}_T(t)$, like here for a similar Fourier sine series. Is this right? – Тyma Gaidash Feb 20 '23 at 00:57
  • @TymaGaidash Yes that wasn't quite right. I think it should have been $\operatorname{\text{Ш}}T(t)=\frac{1}{T}+\frac{2}{T} \underset{N\to\infty}{\text{lim}}\left(\sum\limits{n=1}^N \cos\left(\frac{2 \pi n t}{T}\right)\right)$. – Steven Clark Feb 20 '23 at 04:29